Ask your own question, for FREE!
Mathematics 61 Online
OpenStudy (anonymous):

Improper integral: upper limit: ∞ lower limit: 0. 3e^ -6x dx. The answer says 1, I'm getting -1/2.

Can't find your answer? Make a FREE account and ask your own questions, OR help others and earn volunteer hours!

Join our real-time social learning platform and learn together with your friends!
Can't find your answer? Make a FREE account and ask your own questions, OR help others and earn volunteer hours!

Join our real-time social learning platform and learn together with your friends!